You are on page 1of 6

Section 10.3.

Homework #3 Masaya Sato

Let R be a ring with 1 and M be an R-module.

1. Prove that if A and B are sets of the same cardinality, then the free modules F (A) and
F (B) are isomorphic.
Proof. Let A and B both consist of n elements, i.e.

A = {a1 , . . . , an } and B = {b1 , . . . , bn }.

Then free R-modules generated by A and B are

F (A) = {r1 a1 + · · · + rn an |ri ∈ R} and F (B) = {r1 b1 + · · · + rn bn |ri ∈ R}.

Now define an R-module homomorphism ϕ : F (A) → F (B) by

ϕ(ai ) = bi for i = 1, . . . , n.

Then for every b ∈ F (B) there are r1 , . . . , rn such that

b = r1 b1 + · · · + rn bn .

So for a = r1 a1 + · · · + rn an ∈ F (A)

ϕ(a) = ϕ(r1 a1 + · · · + rn an ) = r1 ϕ(a1 ) + · · · + rn ϕ(an )


= r1 b1 + · · · + rn bn
=b

and thus ϕ is surjective. Moreover for a = r1 a1 + · · · rn an and a0 = r10 a1 + · · · + rn0 an in F (A),


where ri , ri0 ∈ R for i = 1, . . . , n suppose that

ϕ(a) = ϕ(a0 ) ⇒ ϕ(r1 a1 + · · · + rn an ) = ϕ(r10 a1 + · · · + rn0 an )


⇒ r1 ϕ(a1 ) + · · · + rn ϕ(an ) = r10 ϕ(a1 ) + · · · + rn0 ϕ(an ).

This equation reduces to

(r1 − r10 )b1 + · · · + (rn − rn0 )bn = 0.

Since F (B) is a free R-module generated by B, ri = ri0 for i = 1, . . . n. Therefore a = a0 and


hence ϕ is injective. Thus ϕ is an isomorphism and F (A) is isomorphic to F (B).
3. Show that the F [x]-modules in Exercises 18 and 19 of Section 1 are both cyclic.
Proof. Exercise 18. Let V = R2 be the F [x]-module, where F = R, with the linear
transformation  
0 −1
T = .
1 0

Abstract Algebra by Dummit and Foote 1


Section 10.3. Homework #3 Masaya Sato

Then let a = (1, 0)T ∈ V . So for every nonzero v = (r, s)T =∈ V , where r, s ∈ R,

v = (r, s)T = r(1, 0)T + s(0, 1)T = rI(a) + sT (a) = (rI + sT )(a)

and thus V is generated by the single element a ∈ V .


Exercise 19. Next let V = R2 be the F [x]-module, where F = R, with the linear transfor-
mation  
0 0
T =
0 1
and let a = (1, 1) ∈ V . Then for every nonzero v = (r, s)T ∈ V with r, s ∈ R,

v = (r, s)T = (r, r)T + (0, s − r)T = r(1, 1)T + (s − r)(0, 1)T
= rI(a) + (s − r)T (a)
= (rI + (s − r)T )(a).

Therefore V is generated by a and thus V is cyclic.


4. An R-module M is called a torsion module if for each m ∈ M there is a nonzero element
r ∈ R such that rm = 0, where r may depend on m (i.e. M = Tor(M ) in the notation of
Exercise 8 of Section 1). Prove that every finite abelian group is a torsion Z-module. Give
an example of an infinite abelian group that is a torsion Z-module.
Proof. Let G be a finite abelian group. Define a ring action Z × G → G by
k
X
k.g = kg = g
i=1

for all k ∈ Z and all g ∈ G. Then


1.g = 1g = g
and rs r X
s
X X
(rs).g = (rs)g = g= g = r(sg) = r.(s.g)
i=1 i=1 j=1

for all r, s ∈ Z. Therefore for each g ∈ G there exists n ∈ Z>0 such that

ng = ng = g + · · · + g = 0

since g has finite order. Hence n.g = ng = 0 and g is a torsion element. Thus G is a torsion
Z-module.
And an example of infinite abelian group that is a torsion Z-module is Q/Z since every
element a ∈ Q/Z has finite order.
5. Let R be an integral domain. Prove that every finitely generated torsion R-module has
a nonzero annihilator i.e. there is a nonzero element r ∈ R such that rm = 0 for all m ∈ M
– here r does not depend on m (the annihilator of a module was defined in Exercise 9 of
Section 1). Give an example of a torsion R-module whose annihilator is the zero ideal.

Abstract Algebra by Dummit and Foote 2


Section 10.3. Homework #3 Masaya Sato

Proof. Let M be a finitely generated torsion R-module, i.e.

M = {r1 m1 + · · · + rn mn |ri ∈ R ∀i = 1, . . . , n},

where {m1 , . . . , mn } is the generating set. Since each mi is an element in M for i = 1, . . . , n,


there exists nonzero si ∈ R such that

si mi = 0.

Therefore for every m ∈ M has the form

m = r1 m1 + · · · + rn mn

for some ri ∈ R. So r = s1 · · · sn is a nonzero element in R, and

rm = (s1 · · · sn )m = (s1 · · · sn )(r1 m1 + · · · + rn mn )


= ((s1 · · · sn )r1 )m1 + · · · + ((s1 · · · sn )rn )mn
= (s2 · · · sn r1 )(s1 m1 ) + · · · + (s1 · · · sn−1 rn )(sn mn )
= (s2 · · · sn r1 )0 + · · · + (s1 · · · sn−1 rn )0
=0

since R is an integral domain. Hence r is a nonzero annihilator.


For an example of a torsion R-module, where annihilator is the zero ideal, consider a subset
M of the set C(R, R) of all continuous map with compact support. Then M is a module
over itself, where the ring action is given by the pointwise multiplication

f g(x) = f (x)g(x)

for every x ∈ R. Then for every continuous map f ∈ M there exists a nonzero continuous
map g ∈ M so that gf = 0, where g has a distinct compact support from f . However there
is no nonzero g ∈ M such that gf = 0 identically for every f ∈ M .
6. Prove that if M is a finitely generated R-module that is generated by n elements then
every quotient of M may be generated by n (or fewer elements). Deduce that quotients of
cyclic modules are cyclic.
Proof. Let an R-module M be generated by a spanning set A = {a1 , · · · , an }. Then note
that every submodule N is generated by a subset {ai1 , . . . , aik } of A, where {i1 , . . . , ik } ⊂
{1, . . . , n}. So M/N has a spanning set that consists of elements

a1 + N, . . . , an + N ∈ M/N .

However {ai1 , . . . , aik } generates N and thus aij + N = N for j = 1, . . . k. Therefore every
m + N ∈ M/N is of the form

m + N = (r1 a1 + N ) + · · · + (rn an + N ) = r1 a1 + · · · + rn an + N

Abstract Algebra by Dummit and Foote 3


Section 10.3. Homework #3 Masaya Sato

and hence M/N can be generated by n or fewer elements since aij + N = N .


Now suppose that an R-module M is cyclic i.e. there exists an element a ∈ M such that
every m ∈ M is of the form
m = ra
for some r ∈ R. In other words, M is generated by a single element a. This deduces that
every nontrivial quotient module M/N is generated by one element. Therefore M/N is
cyclic.
7. Let N be a submodule of M . Prove that if both M/N and N are finitely generated then
so is M .
Proof. For an R-module M and its submodule N of M , suppose that M/N and N are finitely
generated. Then there exist spanning sets {a1 + N, . . . , am + N } and {b1 , . . . , bn } for M/N
and N , respectively. So for every m ∈ M m + N can be expressed as an R-linear combination
of the spanning set, i.e.
m + N = (r1 a1 + N ) + · · · + (r1 am + N ) = r1 a1 + · · · + rm am + N ,
where r1 , . . . , rm ∈ R and
m − (r1 a1 + · · · + rm am ) ∈ N .
Therefore there are r10 , . . . , rn0 ∈ R such that
m − (r1 a1 + · · · + rm am ) = r10 b1 + · · · + rn0 bn
and hence
m = r1 a1 + · · · + rm am + r10 b1 + · · · + rn0 bn .
Thus M is finitely generated since every m ∈ M can be expressed as an R-linear combination
of the spanning set
{a1 , . . . , am } ∪ {b1 , . . . , bn }.

8. Let S be the collection of sequences (a1 , a2 , a3 , . . . ) of integers a1 , a2 , a3 , . . . where all but


finitely many of the ai are 0 (called the direct sum of infinitely many copies of Z). Recall
that S is a ring under componentwise addition and multiplication and S does not have a
multiplicative identity. Prove that S is not finitely generated as a module over itself.
Proof. Suppose by contradiction that S is finitely generated by n elements of S, i.e.
S = {r1 s1 + · · · + rn sn |ri ∈ Z ∀i = 1, . . . , n}.
Consider the map S 3 si 7→ ki ∈ Z that assigns to the number of nonzero entry of the
sequence si for i = 1, . . . , n. So the number of a nonzero entry s ∈ S is at most
k1 + · · · + kn ,
which is still finite. However this contradicts that S is finitely generated since (2, 2, 2, . . . )
can not be expresses as any linear combination of the generating set {s1 , . . . , sn }. Therefore
S is not finitely generated.

Abstract Algebra by Dummit and Foote 4


Section 10.3. Homework #3 Masaya Sato

9. An R-module M is called irreducible if M 6= 0 and 0 and M are the only submodules


of M . Show that M is irreducible if and only if M 6= 0 and M is cyclic module with any
nonzero element as generator. Determine all the irreducible Z-modules.
Proof. (⇒) Suppose first that an R-module M is irreducible. Since M 6= 0, there is a nonzero
element a ∈ M . Consider a submodule N of M generated by the single element a, i.e.
N = {ra|r ∈ R}.
However N = M since M is irreducible. Thus M is cyclic because of the single generator a.
(⇐) Suppose conversely that M is cyclic. So M has a generator a ∈ M and
M = {ra|r ∈ R}.
Note that 0 and M are both submodules of M . So suppose by contradiction that there is a
proper submodule N of M . Since N is a submodule of M , N is also generated by an element
b ∈ N ≤ M and b = sa for some s ∈ R. However b = sa ∈ M and rb ∈ M implies that
rb = r(sa) ∈ N ,
where r ∈ R. Therefore N = M and this contradicts that N is a proper submodule of M .
Hence M is irreducible.
10. Assume R is commutative. Show that an R-module M is irreducible if and only if M
is isomorphic (as a R-module) to R/I where I is a maximal ideal of R. [By the previous
exercise, if M is irreducible there is a natural map R → M defined by r 7→ rm, where m is
any fixed nonzero element of M .]
Proof. (⇒) Suppose first that M is irreducible. So there exists a generator m ∈ M so that
M = {rm|r ∈ R}.
Define a map ϕ : R → M by
ϕ(r) = rm.
Then ϕ is an R-module homomorphism since for all r, s, t ∈ R, where R is a module over
itself,
ϕ(r + ts) = (r + ts)m = rm + (ts)m = rm + t(sm) = ϕ(r) + tϕ(s).
So the submodule ϕ(R) is 0 or M because M is irreducible. But ϕ(R) 6= 0 because
ϕ(1) = 1m = m 6= 0.
Thus ϕ(R) = M and ϕ(R) is surjective. Now let I = ker ϕ. Then I is a maximal ideal or M
and therefore
R/ ker ϕ ∼= M ⇒ R/I ∼ = M.
(⇐) Conversely suppose that M is isomorphic to R/I, where I is a maximal ideal of R, via
an R-module isomorphism ψ : R/I → M . Recall that R/I is an irreducible quotient module.
Therefore M = ψ(R/I) is also irreducible.

Abstract Algebra by Dummit and Foote 5


Section 10.3. Homework #3 Masaya Sato

11. Show that M1 and M2 are irreducible R-modules, then any nonzero R-module homomor-
phism from M1 to M2 is an isomorphism. Deduce that if M is irreducible then EndR (M )is
a division ring (this result is called Schur’s Lemma). [Consider the kernel and the image.]
Proof. Let ϕ : M1 → M2 be a nonzero R-module isomorphism. Since ϕ is nonzero, im ϕ
is a nonzero submodule of M2 . But im ϕ = M2 because M2 is irreducible and thus does
not contain any proper submodules. So ϕ is surjective. Moreover ker ϕ is a submodule
of M1 . This implies that ker ϕ = M1 or ker ϕ = {0} since M1 is irreducible. So suppose
that ker ϕ = M1 . Then ϕ(M1 ) = {0}, and this is contradiction because ϕ is a nonzero
homomorphism by assumption. Therefore ker ϕ = {0} and hence ϕ is injective.
Then for every R-module homomorphism ψ : M → M , where M is an irreducible R-
module, ψ is an isomorphism by the argument above. I.e. every ψ has the inverse R-module
homomorphism ψ −1 such that

ϕ ◦ ϕ−1 = ϕ−1 ◦ ϕ = idM ,

where idM is the identity R-module homomorphism. Therefore EndR (M ) is a division ring
with the identity homomorphism idM that is distinct from the zero homomorphism.

Abstract Algebra by Dummit and Foote 6

You might also like